1answer.
Ask question
Login Signup
Ask question
All categories
  • English
  • Mathematics
  • Social Studies
  • Business
  • History
  • Health
  • Geography
  • Biology
  • Physics
  • Chemistry
  • Computers and Technology
  • Arts
  • World Languages
  • Spanish
  • French
  • German
  • Advanced Placement (AP)
  • SAT
  • Medicine
  • Law
  • Engineering
Jet001 [13]
3 years ago
15

Raymond spends 1/10 of his vacation budget on a car rental. Write an expression for how much Raymond has to spend on the rest of

his vacation. If he budgets $650 for the vacation, how much will he have left to spend?
A. (1/10) b, $65
B. (9/10) b, $65
C. (9/10) b, $585
D. (1/10) b, $585
Mathematics
1 answer:
Elina [12.6K]3 years ago
4 0

answer
C.

reason
First you must divide, 650/10. Which will equal to 65. Subtract 65 from 650. You will end up with 585. To represent the amount that is left would be 9/10,
You might be interested in
Triangle ABC is equilateral. Find the height of BD
RideAnS [48]

Since ABC is equilateral, all 3 sides have equal length. side AC is 8 units since side BC is 8 units.

Line BD is placed in the middle, making D the midpoint of side AC.

knowing this information we can determine that the length of DC is 4 units (half of AC)

since triangle BDC is a right triangle, we can use the side lengths in the pythagorean theorem to find the length of BD

a²+b²=c² where a & b = legs of triangle , and c= hypotenuse (longest side)

we are given the hypotenuse and found one leg so we can plug our values into the equation to find the third

4² + b²= 8²

16 + b² = 64

b² = 48

b = \sqrt{48}

b= 4√3 or about 6.928 units

hope this helped

3 0
3 years ago
Can someone help me
Morgarella [4.7K]
The total cost would be $4,050. Hope this helps!
7 0
3 years ago
Read 2 more answers
What is 93 times 10 to the sixth power
Rashid [163]
<span>93000000

...........................................</span>
8 0
3 years ago
Read 2 more answers
Which statements<br> are true for the giving geometric sequence check all that apply
professor190 [17]
Where are the statements ???
6 0
3 years ago
What is the greatest value of 2,463.9051
RoseWind [281]

Hey There @Bre18016,

The answer is \boxed{2}

The greatest value of 2,463.9051 would be the thousands place (2) simply as it is the biggest number out of the other places.

For instance, if we had the number 300, 3 would be the greatest value.

Or let's say we had 10,000 the 1 would be the greatest value.

Furthermore, you could look at the first digit in the entire number to deter mine the greatest value.

6 0
3 years ago
Other questions:
  • John had 2/3 of his homework done sarah had 5/10 alex 7/8 and michelle 1/2 who complete most of the homework
    6·2 answers
  • Question is below....
    10·1 answer
  • Last and only one !!!! ((:
    8·2 answers
  • B) Write an expression for fin terms of d.<br> c) Work out e-f<br> Simplify your answer.
    14·1 answer
  • The measure of ac = 7, mbac = 80°, mabc = 50°, the measure of nm = 7, mlnm = 80°, and mnlm = 50°. do the measurements indicate t
    12·1 answer
  • Can someone pleasee help me with the last 4 (5-8)
    7·1 answer
  • the cost to rent skis at a local sporting goods store is $15 plus $20 per day. which equation models the relationship between th
    11·1 answer
  • What is the product of (4i) and (4+i)
    8·2 answers
  • The height of a soccer ball that is kicked from the ground can be approximated by the function:
    13·1 answer
  • Pls answer it pls thanks
    5·2 answers
Add answer
Login
Not registered? Fast signup
Signup
Login Signup
Ask question!